LSAT and Law School Admissions Forum

Get expert LSAT preparation and law school admissions advice from PowerScore Test Preparation.

 Francis O'Rourke
PowerScore Staff
  • PowerScore Staff
  • Posts: 471
  • Joined: Mar 10, 2017
|
#38436
Choice (E) tells us that there has been zero increase in bears in the Abbimac Valley in the past eight years.

Choice (D) may lead you to conclude that there are fewer bears in the Abbimac Valley, but it does not necessarily tell us this. If the population of bears outside the preserve has descreased by, say, 50 bears and the population in the preserve has increased by only 20 bears, then there would have been 30 fewer bears overall.

However, choice (D) also allows for the possibility of more bears overall. If the decrease outside the preserve was only 10 bears and the increase inside the preserve was 20 bears, we would have 10 more bears compared to eight years ago. This possibility would strengthen the argument. Since both possibilities are present, answer choice (D) does not undermine the argument.
 Iam181
  • Posts: 10
  • Joined: Sep 02, 2019
|
#67926
I still do not understand how answer choice E weakens the aurgument.

It is a straight contradition to the premises in the argument. If the preserve's population which holds most of the bears and is within the valley, doubled, then how can it be that valley's population did not increase.

Thank you so much!
 James Finch
PowerScore Staff
  • PowerScore Staff
  • Posts: 943
  • Joined: Sep 06, 2017
|
#68525
Hi Iam181,

It looks like you're missing the scope of the argument by a bit: the premises we have are all about the preserve, which is in a part, but not all of the valley as a whole, although it does contain most/a majority of the bears in the valley. But the conclusion is about the future population of the bears in the valley as a whole, not about the population of the bears in the preserve. So while we know that the preserve's bear population has increased, we don't know how the valley's total bear population has or hasn't changed.

(E) undermines the causal argument in the stimulus that the road will cause the overall valley's bear population to increase by showing the same cause in effect (road closure) and no effect (same bear population). So if this is true, what reason would we have to believe that keeping the road closed will suddenly lead to an increase in the valley as a whole's bear population? There wouldn't be one. This makes (E) correct, as it effectively eliminates the premises about the preserve's bear population from supporting the conclusion about the valley's overall bear population.

Hope this clears things up!

Get the most out of your LSAT Prep Plus subscription.

Analyze and track your performance with our Testing and Analytics Package.